Stay ahead of learning milestones! Enroll in a class over the summer!

G
Topic
First Poster
Last Poster
<DPA+ <AQD =< QIP wanted, incircle circumcircle related
parmenides51   41
N 13 minutes ago by Ilikeminecraft
Source: IMo 2019 SL G6
Let $I$ be the incentre of acute-angled triangle $ABC$. Let the incircle meet $BC, CA$, and $AB$ at $D, E$, and $F,$ respectively. Let line $EF$ intersect the circumcircle of the triangle at $P$ and $Q$, such that $F$ lies between $E$ and $P$. Prove that $\angle DPA + \angle AQD =\angle QIP$.

(Slovakia)
41 replies
parmenides51
Sep 22, 2020
Ilikeminecraft
13 minutes ago
Parallelity and equal angles given, wanted an angle equality
BarisKoyuncu   5
N 22 minutes ago by SleepyGirraffe
Source: 2022 Turkey JBMO TST P4
Given a convex quadrilateral $ABCD$ such that $m(\widehat{ABC})=m(\widehat{BCD})$. The lines $AD$ and $BC$ intersect at a point $P$ and the line passing through $P$ which is parallel to $AB$, intersects $BD$ at $T$. Prove that
$$m(\widehat{ACB})=m(\widehat{PCT})$$
5 replies
BarisKoyuncu
Mar 15, 2022
SleepyGirraffe
22 minutes ago
Cyclic points and concurrency [1st Lemoine circle]
shobber   10
N an hour ago by Ilikeminecraft
Source: China TST 2005
Let $\omega$ be the circumcircle of acute triangle $ABC$. Two tangents of $\omega$ from $B$ and $C$ intersect at $P$, $AP$ and $BC$ intersect at $D$. Point $E$, $F$ are on $AC$ and $AB$ such that $DE \parallel BA$ and $DF \parallel CA$.
(1) Prove that $F,B,C,E$ are concyclic.

(2) Denote $A_{1}$ the centre of the circle passing through $F,B,C,E$. $B_{1}$, $C_{1}$ are difined similarly. Prove that $AA_{1}$, $BB_{1}$, $CC_{1}$ are concurrent.
10 replies
shobber
Jun 27, 2006
Ilikeminecraft
an hour ago
Hard functional equation
Jessey   4
N 2 hours ago by jasperE3
Source: Belarus 2005
Find all functions $f:N -$> $N$ that satisfy $f(m-n+f(n)) = f(m)+f(n)$, for all $m, n$$N$.
4 replies
Jessey
Mar 11, 2020
jasperE3
2 hours ago
Vertices of a convex polygon if and only if m(S) = f(n)
orl   12
N 2 hours ago by Maximilian113
Source: IMO Shortlist 2000, C3
Let $ n \geq 4$ be a fixed positive integer. Given a set $ S = \{P_1, P_2, \ldots, P_n\}$ of $ n$ points in the plane such that no three are collinear and no four concyclic, let $ a_t,$ $ 1 \leq t \leq n,$ be the number of circles $ P_iP_jP_k$ that contain $ P_t$ in their interior, and let \[m(S)=a_1+a_2+\cdots + a_n.\]Prove that there exists a positive integer $ f(n),$ depending only on $ n,$ such that the points of $ S$ are the vertices of a convex polygon if and only if $ m(S) = f(n).$
12 replies
orl
Aug 10, 2008
Maximilian113
2 hours ago
Imo Shortlist Problem
Lopes   35
N 2 hours ago by Maximilian113
Source: IMO Shortlist 2000, Problem N4
Find all triplets of positive integers $ (a,m,n)$ such that $ a^m + 1 \mid (a + 1)^n$.
35 replies
Lopes
Feb 27, 2005
Maximilian113
2 hours ago
Inspired by Humberto_Filho
sqing   0
3 hours ago
Source: Own
Let $ a,b\geq 0 $ and $a + b \leq 2$. Prove that
$$\frac{a^2+1}{(( a+ b)^2+1)^2} \geq  \frac{1}{25} $$$$\frac{(a^2+1)(b^2+1)}{((a+b)^2+1)^2} \geq  \frac{4}{25} $$$$ \frac{a^2+1}{(( a+ 2b)^2+1)^2} \geq  \frac{1}{289} $$$$ \frac{a^2+1}{((2a+ b)^2+1)^2} \geq  \frac{5}{289} $$


0 replies
sqing
3 hours ago
0 replies
Inequalities
Scientist10   2
N 3 hours ago by arqady
If $x, y, z \in \mathbb{R}$, then prove that the following inequality holds:
\[
\sum_{\text{cyc}} \sqrt{1 + \left(x\sqrt{1 + y^2} + y\sqrt{1 + x^2}\right)^2} \geq \sum_{\text{cyc}} xy + 2\sum_{\text{cyc}} x
\]
2 replies
Scientist10
Yesterday at 6:36 PM
arqady
3 hours ago
$n$ with $2000$ divisors divides $2^n+1$ (IMO 2000)
Valentin Vornicu   65
N 3 hours ago by ray66
Source: IMO 2000, Problem 5, IMO Shortlist 2000, Problem N3
Does there exist a positive integer $ n$ such that $ n$ has exactly 2000 prime divisors and $ n$ divides $ 2^n + 1$?
65 replies
Valentin Vornicu
Oct 24, 2005
ray66
3 hours ago
Find the smallest of sum of elements
hlminh   0
3 hours ago
Let $S=\{1,2,...,2014\}$ and $X=\{a_1,a_2,...,a_{30}\}$ is a subset of $S$ such that if $a,b\in X,a+b\leq 2014$ then $a+b\in X.$ Find the smallest of $\dfrac{a_1+a_2+\cdots+a_{30}}{30}.$
0 replies
hlminh
3 hours ago
0 replies
Easy IMO 2023 NT
799786   133
N 3 hours ago by Maximilian113
Source: IMO 2023 P1
Determine all composite integers $n>1$ that satisfy the following property: if $d_1$, $d_2$, $\ldots$, $d_k$ are all the positive divisors of $n$ with $1 = d_1 < d_2 < \cdots < d_k = n$, then $d_i$ divides $d_{i+1} + d_{i+2}$ for every $1 \leq i \leq k - 2$.
133 replies
799786
Jul 8, 2023
Maximilian113
3 hours ago
Complicated FE
XAN4   2
N 3 hours ago by cazanova19921
Source: own
Find all solutions for the functional equation $f(xyz)+\sum_{cyc}f(\frac{yz}x)=f(x)\cdot f(y)\cdot f(z)$, in which $f$: $\mathbb R^+\rightarrow\mathbb R^+$
Note: the solution is actually quite obvious - $f(x)=x^n+\frac1{x^n}$, but the proof is important.
Note 2: it is likely that the result can be generalized into a more advanced questions, potentially involving more bash.
2 replies
XAN4
Yesterday at 11:53 AM
cazanova19921
3 hours ago
Cute diophantine
TestX01   0
4 hours ago
Find all sequences of four consecutive integers such that twice their product is perfect square minus nine.
0 replies
TestX01
4 hours ago
0 replies
A point on the midline of BC.
EmersonSoriano   5
N Apr 6, 2025 by ehuseyinyigit
Source: 2017 Peru Southern Cone TST P5
Let $ABC$ be an acute triangle with circumcenter $O$. Draw altitude $BQ$, with $Q$ on side $AC$. The parallel line to $OC$ passing through $Q$ intersects line $BO$ at point $X$. Prove that point $X$ and the midpoints of sides $AB$ and $AC$ are collinear.
5 replies
EmersonSoriano
Apr 5, 2025
ehuseyinyigit
Apr 6, 2025
A point on the midline of BC.
G H J
G H BBookmark kLocked kLocked NReply
Source: 2017 Peru Southern Cone TST P5
The post below has been deleted. Click to close.
This post has been deleted. Click here to see post.
EmersonSoriano
44 posts
#1 • 3 Y
Y by PikaPika999, KAME06, RANDOM__USER
Let $ABC$ be an acute triangle with circumcenter $O$. Draw altitude $BQ$, with $Q$ on side $AC$. The parallel line to $OC$ passing through $Q$ intersects line $BO$ at point $X$. Prove that point $X$ and the midpoints of sides $AB$ and $AC$ are collinear.
This post has been edited 1 time. Last edited by EmersonSoriano, Apr 5, 2025, 7:33 PM
Reason: change
Z K Y
The post below has been deleted. Click to close.
This post has been deleted. Click here to see post.
RANDOM__USER
7 posts
#2 • 1 Y
Y by PikaPika999
Let \(A\) move projectively on the circumcircle of \(ABC\). Then \(BO\) and \(CO\) remain constant and \(Q\) moves projectively as it is a projection of \(B\) onto a projectively rotating line \(AC\). Now let us consider the point \(\infty_{CO}\) (meaning the point at infinity through which \(OC\) passes), then \(Q\infty_{CO}\) is a projectively moving line, then intersect that line with the constant line \(BO\) to obtain a projectively moving point \(X\). Points \(D\) and \(E\) obviously move projectively. Then essentially, we have the following configuration of projective transformations,

\(A \rightarrow Q \rightarrow \mathcal{P}(\infty_{CO}) \rightarrow BO \rightarrow \mathcal{P}(\infty_{BC})\)

The last step is to obtain the line through \(X\) which is supposed to be the middle line of the triangle. Then we simply consider another projective transformation which maps \(A\) to the midline. All that is left is to prove that both of these transformations are equal, to do this we simply need to consider three convinient positions for \(A\).

Case 1 and 2: If \(AB \perp BC\), then the statement is obvious because \(X\) is the midpoint of \(AC\). The same for \(AC \perp BC\).
Case 3: When \(AB = BC\) the statement is again trivial, due to \(X = Q = E\), where \(E\) is the midpoint of \(AC\).

Thus the two projective transformations are equal and thus \(X\) always lies on the midline of \(\triangle ABC\).
Z K Y
The post below has been deleted. Click to close.
This post has been deleted. Click here to see post.
hukilau17
283 posts
#3
Y by
Quick complex bash: let $\triangle ABC$ be inscribed in the unit circle so that
$$|a|=|b|=|c|=1$$$$o=0$$$$q=\frac{b^2+ab+bc-ac}{2b}$$Now since $X$ is on line $BO$, we have
$$\overline{x} = \frac{x}{b^2}$$And since $QX\parallel CO$, we have
$$\frac{q-x}{c-o} \in \mathbb{R}$$$$\frac{b^2+ab+bc-ac-2bx}{2bc} = \frac{abc+b^2c+ab^2-b^3-2acx}{2ab^2}$$$$ab^3+a^2b^2+ab^2c-a^2bc-2ab^2x = abc^2+b^2c^2+ab^2c-b^3c-2ac^2x$$$$x = \frac{a^2b^2-a^2bc+ab^3-abc^2+b^3c-b^2c^2}{2ab^2-2ac^2} = \frac{a^2b+ab^2+abc+b^2c}{2ab+2ac} = \frac{b(a+b)(a+c)}{2a(b+c)}$$Then
$$\frac{a+b}2 - x = \frac{a(a+b)(b+c) - b(a+b)(a+c)}{2a(b+c)} = \frac{c(a-b)(a+b)}{2a(b+c)}$$and so
$$\frac{\frac{a+b}2-x}{\frac{a+b}2-\frac{a+c}2} = \frac{c(a-b)(a+b)}{a(b+c)(b-c)}$$which is real. $\blacksquare$
Z K Y
The post below has been deleted. Click to close.
This post has been deleted. Click here to see post.
KAME06
154 posts
#4
Y by
Let $M$ and $N$ the midpoints of $AB$ and $AC$, respectively; $Y \in OM \cap BQ$ and $\angle OCB=\alpha$
(1)We know that $\triangle BOC$ is isosceles, so $\angle BOC= 180^\circ -2\alpha$. Also, using the Central Angle Theorem: $\angle BAC = \angle MAQ= \angle BAQ= \frac{180^\circ -2\alpha}{2}=90^\circ -\alpha$.
(2)$90^\circ = \angle BQA \Rightarrow \angle YBA = \angle QBA = 180^\circ - \angle BQA - \angle BAQ=180^\circ - 90^\circ +\alpha-90^\circ=\alpha$.
(3)$\angle YAQ = \angle BAC - \angle MAY= 90^\circ -\alpha -\alpha = 90^\circ -2\alpha$ ($\angle MAY = \alpha$ because is well known that $OM$ is the perpendicular bisector of $BA$, so also $YM$ and $\triangle YBA$ is isosceles).
(4)$\angle YMA = \angle YQA = 90^\circ$, so $AMYQ$ must be cyclic. Then, $\angle YAQ= \angle YMQ = 90^\circ -2\alpha$.
(5)$\angle QMA = \angle YMA - \angle YMQ=90^\circ -90^\circ +2\alpha=2\alpha$
(6)We know that $QX \parallel OC$, so $\angle QXB=180^\circ -\angle BOC=180^\circ-180^\circ+2\alpha=2\alpha$.
(7)Using (5) and (6), we deduce that $MQXB$ must be cyclic (supplementary opposite angles). Then $\alpha = \angle MAY = \angle MQY = \angle MQB = \angle MXB$
(8)$\angle MXB = \alpha = \angle OBC = \angle XBC$, so $MX \parallel BC$, but $MN \parallel BC$, so $X$ and the midpoints of sides $AB$ and $AC$ are collinear,
This post has been edited 1 time. Last edited by KAME06, Apr 5, 2025, 10:32 PM
Z K Y
The post below has been deleted. Click to close.
This post has been deleted. Click here to see post.
ehuseyinyigit
810 posts
#6
Y by
Just one ratio and it finishes :)

Let midpoints of $AB$ and $AC$ be $M_C$ and $M_B$ respectively. Then
$$\dfrac{TO}{OB}=\dfrac{TM_B}{M_BQ}=\dfrac{TO}{OC}=\dfrac{TX}{TQ}$$implies line $M_BX$ bisects $\angle QXB$. Thus $\angle QXM_B=\angle OCB$. Since $QX\parallel OC$ we also have $\angle M_BQX=M_BCO$. Thus, $X-M_B-M_C$ as desired.
Z K Y
The post below has been deleted. Click to close.
This post has been deleted. Click here to see post.
ehuseyinyigit
810 posts
#7 • 1 Y
Y by teomihai
By the way, an interesting property about the problem is:

Let $B'$ be antipode of the point $B$. Then $AM_BXB'$ is cyclic. Also we have $\triangle ABH\sim \triangle QXM_B$.
Z K Y
N Quick Reply
G
H
=
a